Chuyên đề Phần nguyên - Số học

#1

Đã gửi 21-07-2011 - 10:44

Zaraki

Bạn đang xem: Chuyên đề Phần nguyên - Số học

    PQT

  • Phó Quản lý Toán Cao cấp
  • 4273 Bài viết

PHẦN NGUYÊN CỦA MỘT SỐ THỰC

A. ĐỊNH NGHĨA

Ta hiểu được, từng số thực $x$ đều hoàn toàn có thể viết lách được bên dưới dạng

$x=n+z$

trong ê $n$ là số vẹn toàn và $0 \le z \le 1$

Chẳng hạn:

$7,9=7+0,9$
$-7,9=-8+0,1$

Hơn nữa, cơ hội viết lách như bên trên là có một không hai. Ta gọi số vẹn toàn $n$ là phần vẹn toàn của $x$ và kí hiệu là $[x]$; còn $z$ được gọi là phần phân của $x$ và ê hiệu là $\left \{ x \right \}$.

Từ phân tách, tao rút đi ra quyết định nghĩa
Định nghĩa: Phần vẹn toàn của số thực $x$, kí hiệu là $[x]$, là số vẹn toàn lớn số 1 ko vượt lên trước vượt $x$. Phần phân của số thực $x$ được khái niệm vì thế $\left \{ x \right \}=x-[x]$.

Ngoài cơ hội gọi thường thì là phần vẹn toàn (intergal part) của $x$ với kí hiệu là $[x]$, một số trong những người sáng tác quốc tế còn gọi này là floor function và kí hiệu là $\left \lfloor x \right \rfloor$. Sở dĩ thế vì thế người tao nêu ro ceiling function - kí hiệu $\left \lceil x \right \rceil$, như khái niệm sau đây

$\left \lceil x \right \rceil$ là số vẹn toàn nhỏ nhất vượt lên trước vượt $x$

Dễ dàng thấy rằng

$\left \lceil x \right \rceil=\left\{\begin{matrix}x=\left \lfloor x \right \rfloor ; x\in \mathbb{Z} & & \\ \left \lceil x \right \rceil+1 ; x\notin \mathbb{Z} & & \end{matrix}\right.$

B. TÍNH CHẤT

1) $x=[x]+\left \{ x \right \}$
2) $x=[x] \Leftrightarrow x \in \mathbb{Z}$
3) $x=\left \{ x \right \} \Leftrightarrow 0\le x < 1$
4) $x-1<[x] \le x$
5) Nếu $k$ vẹn toàn thì

$[x+k]=[x]+k$ và $\left \{ x+k \right \}=\left \{ x \right \}+k$

Bạn hãy luyện chứng tỏ những đặc thù này đi!

Xin trả thêm thắt một số trong những tính chất
6) $[x+y] \ge [x]+[y]$
7) $[x] \le x <[x]+1$
8) Nếu $x \ge y$ thì $[x] \ge [y]$

9) $0 \le \left \{ x \right \} <1$
10) $\left \{ x+y \right \} \le \left \{ x \right \} + \left \{ nó \right \}$

Chứng minh đặc thù 6
Viết $x=[x]+\left \{ x \right \}, y=[y]+\left \{ nó \right \}$
Khi đó
$[x+y]=[([x]+[y])+(\left \{ x \right \}+\left \{ nó \right \})]=[x]+[y]+[\left \{ x \right \}+\left \{ nó \right \}]$. (1)
Vì $\left \{ x \right \} \ge 0$ và $\left \{ nó \right \} \ge 0$ nên $[\left \{ x \right \}+\left \{ nó \right \}] \ge 0$.
Kết phù hợp với (1) tao suy ra

$[x+y] \ge [x]+[y]$

Chứng minh đặc thù 8
Vì $x \ge y$ nên $\exists \alpha \ge 0$ sao cho:

$x=y+\alpha$ hoặc $x=[y]+(\left \{ nó \right \}+\alpha)$.

Suy đi ra $[x]=[y]+[(\left \{ nó \right \}+\alpha)]$. (1)
Vì $\alpha \ge 0$ và $\left \{ nó \right \} \ge 0$ nên $\left \{ nó \right \}+\alpha \ge 0$ và $[(\left \{ nó \right \}+ \alpha)] \ge 0$.
Kết phù hợp với (1) tao với $[x] \ge [y]$.

Xin ra mắt thêm thắt một số trong những đặc thù khá là thú vị
1) Giả sử $0<\alpha \in \mathbb{R}$ và $n \in \mathbb{N}$. Lúc ê $\left [ \dfrac{\alpha}{n} \right ]$ là số toàn bộ những số vẹn toàn dương là bội của $n$ tuy nhiên ko vượt lên trước vượt $\alpha$.
2) Giả sử $0<\alpha \in \mathbb{R}$ và $n \in \mathbb{N}$. Lúc ê,

$\left [ \dfrac{n}{\alpha} \right ]$

là số toàn bộ những số vẹn toàn dương là bội của $\alpha$ tuy nhiên ko vượt lên trước vượt $n$.
3) Nếu $a$ và $b$ là nhị số ko âm, thì

$[2a]+[2b] \ge [a]+[b] + [a+b]$

Bài viết lách và được sửa đổi nội dung vì thế E. Galois: 19-08-2012 - 08:56

Discovery is a child’s privilege. I mean the small child, the child who is not afraid đồ sộ be wrong, đồ sộ look silly, đồ sộ not be serious, and đồ sộ act differently from everyone else. He is also not afraid that the things he is interested in are in bad taste or turn out đồ sộ be different from his expectations, from what they should be, or rather he is not afraid of what they actually are. He ignores the silent and flawless consensus that is part of the air we breathe – the consensus of all the people who are, or are reputed đồ sộ be, reasonable.

Grothendieck, Récoltes et Semailles (“Crops and Seeds”). 


#2

Đã gửi 27-07-2011 - 09:37

NguyThang khtn

    Thượng úy

  • Hiệp sỹ
  • 1468 Bài viết

Đây là một số trong những tư liệu của thầy hxthanh :
Chuyên đề phần nguyên

It is difficult đồ sộ say what is impossible, for the dream of yesterday is the hope of today and the reality of tomorrow


#3

Đã gửi 02-08-2011 - 15:09

Zaraki

    PQT

  • Phó Quản lý Toán Cao cấp
  • 4273 Bài viết

Anh Thắng hoàn toàn có thể post một số trong những đặc thù nữa về phần vẹn toàn lên phía trên được ko anh? Cảm ơn anh nhiều!

Discovery is a child’s privilege. I mean the small child, the child who is not afraid đồ sộ be wrong, đồ sộ look silly, đồ sộ not be serious, and đồ sộ act differently from everyone else. He is also not afraid that the things he is interested in are in bad taste or turn out đồ sộ be different from his expectations, from what they should be, or rather he is not afraid of what they actually are. He ignores the silent and flawless consensus that is part of the air we breathe – the consensus of all the people who are, or are reputed đồ sộ be, reasonable.

Grothendieck, Récoltes et Semailles (“Crops and Seeds”). 


#4

Đã gửi 09-08-2011 - 12:18

hxthanh

    Tín vật $\sum$

  • Hiệp sỹ
  • 3921 Bài viết

Bạn Phạm Quang Toàn thân mật mến!

Tôi cực kỳ vui sướng vì thế các bạn cũng tương đối quan hoài cho tới chủ thể này
Phần Nguyên là 1 trong những chủ thể cực kỳ khó khăn, nhưng mà giáo trình tất cả chúng ta được học tập lại không tồn tại nhiều.
Trong thật nhiều Việc số học tập, vô tình tao vẫn sử dụng cho tới kiến thức và kỹ năng của hàm này nhưng mà nhập cơ hội tư duy nhưng mà không hiểu biết nhiều được xuất xứ của chính nó.
Một lịch trình ra mắt không thiếu thốn về Phần Nguyên, lúc bấy giờ vẫn tồn tại là 1 trong những chủ đề hé.
Vì vậy, tôi, các bạn và toàn bộ tất cả chúng ta tiếp tục cùng với nhau trao thay đổi những kiến thức và kỹ năng vẫn biết, những Việc vẫn thưởng thức hoặc những gì nhưng mà chúng ta vẫn nghiên cứu và phân tích được về Phần Nguyên nhằm toàn bộ người xem xem thêm.

Dưới phía trên, tôi nài thể hiện một số trong những bài xích tập:

1. Tìm độ quý hiếm lớn số 1 và nhỏ nhất của
$P=\dfrac{2x\left\lfloor 3x \right\rfloor -x^2+4}{{\left\lfloor x \right\rfloor}^2+2x^2+1}$

2. Biểu thao diễn sản phẩm số sau vì thế một công thức duy nhất!
$1,1,2,3,4,9,8,27,...,2^{n-1},3^{n-1},...$

3. Chứng minh rằng với từng n vẹn toàn dương tao có:
$\left\lfloor\sqrt{n}+\dfrac{1}{2}\right\rfloor+\left\lfloor\sqrt{n-1}\right\rfloor=\left\lfloor\sqrt{4n-3}\right\rfloor$

Tạm thời thế vẫn :icon6:

Bài viết lách và được sửa đổi nội dung vì thế hxthanh: 02-11-2011 - 07:26


#5

Đã gửi 01-11-2011 - 20:01

Zaraki

    PQT

  • Phó Quản lý Toán Cao cấp
  • 4273 Bài viết

Đang do dự một số trong những bài xích, nài trả lên

1. Tìm số bất ngờ $k$ lớn số 1 sao cho
$$(1994!)^{1995} \ \vdots \ 1995^k$$
2. Có từng nào số bất ngờ $x$ thỏa mãn
$$\left[ \dfrac{x}{2010} \right] = \left[ \dfrac{x}{2011} \right] = \left[ \dfrac{x}{2012} \right]$$
3. Tìm $x$ biết
$$ \left[ -x^2+3x \right] = \left[ x^2+ \dfrac{1}{2} \right]$$

4. Tìm $x$ biết
$$[x]^2-[x]-2=0$$

Bài viết lách và được sửa đổi nội dung vì thế Phạm Quang Toàn: 01-11-2011 - 20:04

Discovery is a child’s privilege. I mean the small child, the child who is not afraid đồ sộ be wrong, đồ sộ look silly, đồ sộ not be serious, and đồ sộ act differently from everyone else. He is also not afraid that the things he is interested in are in bad taste or turn out đồ sộ be different from his expectations, from what they should be, or rather he is not afraid of what they actually are. He ignores the silent and flawless consensus that is part of the air we breathe – the consensus of all the people who are, or are reputed đồ sộ be, reasonable.

Grothendieck, Récoltes et Semailles (“Crops and Seeds”). 


#6

Đã gửi 02-11-2011 - 00:28

hxthanh

    Tín vật $\sum$

  • Hiệp sỹ
  • 3921 Bài viết

Bài 1: Sử dụng quyết định lý Lagrande về số nón của tối đa của một số trong những thành phần chứa chấp nhập n!

Ta với $1995=3.5.7.19$

Theo định lý Legendre thì số nón tối đa của $19$ với nhập $(1994)!$ là:

$\left\lfloor\dfrac{1994}{19}\right\rfloor+\left\lfloor\dfrac{1994}{(19)^2}\right\rfloor+...+\left\lfloor\dfrac{1994}{(19)^k}\right\rfloor+...=109$

Như vậy $(1994)!\;\vdots\; (1995)^{109}$ và $(1994)!\;\not{\vdots} \;(1995)^{n\ge110}$

Suy đi ra nhằm $((1994)!)^{1995}\;\vdots\; (1995)^k$ thì $k\le 109*1995=217\;455$


#7

Đã gửi 02-11-2011 - 01:17

hxthanh

    Tín vật $\sum$

  • Hiệp sỹ
  • 3921 Bài viết

Bài 2

Xét hệ phương trình: $\left\lfloor\dfrac{x}{2010}\right\rfloor=\left\lfloor\dfrac{x}{2011}\right\rfloor=\left\lfloor\dfrac{x}{2012}\right\rfloor$

Vì $x\in\mathbb{N}$ nên tao hoàn toàn có thể đặt điều $x=2010k+r,\;\;\;(0\le k\in\mathbb{N};\;0\le r \le 2009$

Thay nhập hệ bên trên tao có:

$\left\lfloor\dfrac{2010k+r}{2010}\right\rfloor=\left\lfloor\dfrac{2011k+r-k}{2011}\right\rfloor=\left\lfloor\dfrac{2012k+r-2k}{2012}\right\rfloor$

$\Leftrightarrow k=k+\left\lfloor\dfrac{r-k}{2011}\right\rfloor=k+\left\lfloor\dfrac{r-2k}{2012}\right\rfloor$

$\Leftrightarrow \left\lfloor\dfrac{r-k}{2011}\right\rfloor=\left\lfloor\dfrac{r-2k}{2012}\right\rfloor=0$

Suy ra: $r-2k\ge 0 \Rightarrow 2k\le r\le 2009\Rightarrow 0\le k\le 1004$

Vậy với $1005$ độ quý hiếm của k (từ $0$ cho tới $1004$). Tương ứng với từng độ quý hiếm $k$ thì $r$ nhận những độ quý hiếm kể từ $2k$ cho tới $2009$, suy đi ra với $2009-2k+1=2010-2k$ độ quý hiếm của $r$ ứng với từng độ quý hiếm của $k$

Như vậy số nghiệm bất ngờ của hệ bên trên là: $\sum\limits_{k=0}^{1004} (2010-2k)=1\;011\;030$ (em tính lại coi đúng không nhỉ nhé!)


#8

Đã gửi 02-11-2011 - 02:28

hxthanh

    Tín vật $\sum$

  • Hiệp sỹ
  • 3921 Bài viết

Bài 3: Giải phương trình:

$\left\lfloor-x^2+3x\right\rfloor=\left\lfloor x^2+\dfrac{1}{2}\right\rfloor$

Lời giải:
Đặt độ quý hiếm phần vẹn toàn của 2 vế là $m$, tao có:
$m=\left\lfloor x^2+\dfrac{1}{2}\right\rfloor \ge \left\lfloor \dfrac{1}{2}\right\rfloor=0\Rightarrow m\ge 0$
Suy ra:
$-x^2+3x\ge 0 \Rightarrow 0\le x \le 3$.
Mặt khác: Theo bất đẳng thức AM-GM thì $x(3-x)\le (\dfrac{x+3-x}{2})^2=\dfrac{9}{4}<3$
Do đó: $m\in \{0,1,2\}$
Từ phía trên tao với 3 hệ bất phương trình ứng với 3 độ quý hiếm của $m$ là:

$(I)\;\;\;\begin{cases}0\le -x^2+3x <1 \\ 0\le x^2+\dfrac{1}{2}<1\end{cases}\;\;\;(II)\;\;\;\begin{cases}1\le -x^2+3x <2 \\ 1\le x^2+\dfrac{1}{2}<2\end{cases}\;\;\;\text{và}\;\;\;(III)\;\;\;\begin{cases}2\le -x^2+3x <3 \\ 2\le x^2+\dfrac{1}{2}<3\end{cases}$

Nghiệm của (I) là $0\le x <\dfrac{3-\sqrt 5}{2}$

Nghiệm của (II) là $\dfrac{\sqrt2}{2}\le x<1$

Nghiệm của (III) là $\sqrt{\dfrac{3}{2}}\le x<\sqrt{\dfrac{5}{2}}$

(Đề nghị độc giả tự động giải và kiểm tra)

Nghiệm của phương trình vẫn nghĩ rằng hợp ý của 3 khoảng tầm bên trên $x\in \left[0,\dfrac{3-\sqrt 5}{2}\right)\cup\left[\dfrac{\sqrt2}{2},1\right)\cup\left[\sqrt{\dfrac{3}{2}},\sqrt{\dfrac{5}{2}}\right)$


#9

Đã gửi 02-11-2011 - 02:41

hxthanh

    Tín vật $\sum$

  • Hiệp sỹ
  • 3921 Bài viết

Bài 4: (Rất đơn giản)
Giải phương trình: $\lfloor x \rfloor^2-\lfloor x \rfloor-2=0$

Ta có: $\lfloor x \rfloor^2-\lfloor x \rfloor-2=\left(\lfloor x\rfloor +1\right)\left(\lfloor x\rfloor -2\right)=0$
Từ ê suy ra:
hoặc $\lfloor x \rfloor=-1\Leftrightarrow -1\le x<0$
hoặc $\lfloor x \rfloor=2\Leftrightarrow 2\le x<3$
--------
HẾT!


#10

Đã gửi 07-02-2012 - 16:17

hxthanh

    Tín vật $\sum$

  • Hiệp sỹ
  • 3921 Bài viết

Một số bài xích luyện tự động luyện nhập mục chính phần nguyên

$\boxed{Bt1.6}$
Cho $A=\sqrt{4n^2+n},\; n\in\mathbb{N}$. Chứng minh rằng: $\{A\}\le \dfrac{1}{4}$

(Romania - 2003)

$\boxed{Bt1.7}$
Chứng minh rằng: $\left\lfloor 5x \right\rfloor + \left\lfloor 5y \right\rfloor \ge \left\lfloor 3x+y \right\rfloor + \left\lfloor x+3y \right\rfloor+ \left\lfloor x \right\rfloor + \left\lfloor nó \right\rfloor$
Từ thành quả ê chứng tỏ $(5m)!(5n)!$ phân chia không còn cho tới $m!n!(3m+n)!(3n+m)!$

Xem thêm: Linh Kiện Thành Công - Linh kiện chất lượng - giá cả hợp lý

(USAMO-1975)

$\boxed{Bt1.8}$
Tìm số bất ngờ $n$ nhỏ nhất sao cho tới $n!$ tận nằm trong vì thế $290$ chữ số $0$

(HMMT-2003)

$\boxed{Bt1.10}$

Tính tổng: $S=\sum\limits_{k=0}^{2009}\left(\left\lfloor \dfrac{3^k+2010}{3^{k+1}} \right\rfloor + \left\lfloor \dfrac{2010-3^k}{3^{k+1}} \right\rfloor \right)$

$\boxed{Bt1.11}$
Chứng minh rằng: $\sum\limits_{k=0}^{+\infty} \left\lfloor \dfrac{x+2^k}{2^{k+1}}\right\rfloor = \left\lfloor x \right\rfloor$

$\boxed{Bt2.4}$

Tính tổng $S_n=\sum\limits_{k=1}^n \left\lfloor \sqrt{k}+ \dfrac{1}{2} \right\rfloor$

$\boxed{Bt2.5}$

Tính tổng $S_n=\sum\limits_{k=1}^n \left\lfloor 2\sqrt{k} \right\rfloor$

$\boxed{Bt2.6}$
Cho sản phẩm số $\{U_n\}_1^\infty \; :\;\{1,2,2,2,3,3,3,3,3,4,4,4,4,4,4,4,5,...\}$
Được xác lập vì thế quy luật: $1$ số $1$; $3$ số $2$; $5$ số $3$;...;$2k-1$ số $k$;...
Tìm số hạng tổng quát tháo của sản phẩm số bên trên.

$\boxed{Bt2.11}$

Tính: $S=\sum\limits_{k=1}^n \left\lfloor \dfrac{k^2-3k+2}{5} \right\rfloor$

$\boxed{Bt2.12}$

Tính $S=\sum\limits_{k=1}^{n-1}\left\{ \dfrac{km}{n} \right\}$, với $m,n \in\mathbb{N}^*;\;n\ge 2$

(JMO-1995)

$\boxed{Bt2.13}$
Cho $\lambda$ là một số trong những vô tỷ dương, $n$ là một số trong những vẹn toàn dương. Chứng minh rằng:

$\sum\limits_{k=1}^n \left\lfloor k\lambda \right\rfloor + \sum\limits_{k=1}^{\left\lfloor n\lambda \right\rfloor} \left\lfloor \dfrac{k}{\lambda} \right\rfloor = n\left\lfloor n\lambda \right\rfloor$

$\boxed{Bt2.14}$

Tính $S=\sum\limits_{k=1}^{\frac{n(n+1)}{2}} \left\lfloor \dfrac{\sqrt{8k+1}-1}{2} \right\rfloor$

$\boxed{Bt2.18}$
Cho $m,n$ là những số vẹn toàn dương.
Tính $S=\sum\limits_{k=1}^n \left\lfloor \dfrac{m}{k} \right\rfloor$

$\boxed{Bt2.19}$
Cho $p$ là số thành phần lẻ, $q$ là số vẹn toàn ko phân chia không còn cho tới $p$.
Chứng minh rằng:
$\sum\limits_{k=1}^{p-1} \left\lfloor (-1)^k k^2\dfrac{q}{p}\right\rfloor = \dfrac{(p-1)(q-1)}{2}$

$\boxed{Bt2.20}$
Cho $p$ là số thành phần lẻ,
Chứng minh rằng:
$\sum\limits_{k=1}^{p-1} \dfrac{k^p-k}{p} \equiv \dfrac{p+1}{2} \;\pmod{p}$

$\boxed{Bt2.21}$
Cho $p$ và $q$ là 2 số lẻ,
Tính độ quý hiếm biểu thức:

$A=\sum\limits_{k=1}^{\frac{p-1}{2}} \left\lfloor \dfrac{kq}{p} \right\rfloor + \sum\limits_{k=1}^{\frac{q-1}{2}} \left\lfloor \dfrac{kp}{q} \right\rfloor$

$\boxed{Bt2.22}$
Cho số vẹn toàn $n\ge 2$

Tính: $S=\sum\limits_{m=1}^{ \left\lfloor \frac{n}{2} \right\rfloor } \sum\limits_{k=1}^{n+1-2m} \left\lfloor \dfrac{n-m}{k+m-1} \right\rfloor$
__________________________________


#11

Đã gửi 11-02-2012 - 11:23

E. Galois

    Chú lùn loại 8

  • Quản lý Toán Phổ thông
  • 3861 Bài viết

Đề nghị anh Thanh và Toàn sớm hoàn thiện mục chính này nhằm đăng tải trang chủ


#12

Đã gửi 12-02-2012 - 20:05

tieulyly1995

    Sĩ quan

  • Thành viên
  • 435 Bài viết

Một số bài xích luyện tự động luyện nhập mục chính phần nguyên

$\boxed{Bt1.7}$
Chứng minh rằng: $\left\lfloor 5x \right\rfloor + \left\lfloor 5y \right\rfloor \ge \left\lfloor 3x+y \right\rfloor + \left\lfloor x+3y \right\rfloor+ \left\lfloor x \right\rfloor + \left\lfloor nó \right\rfloor$
Từ thành quả ê chứng tỏ $(5m)!(5n)!$ phân chia không còn cho tới $m!n!(3m+n)!(3n+m)!$

(USAMO-1975)

__________________________________

Bài này thầy ghi sai đề rồi, nhập phần tự động luyện của thầy là thế này nhưng lại : :closedeyes:
CMR :
$\left\lfloor 5x \right\rfloor + \left\lfloor 5y \right\rfloor \ge \left\lfloor 3x+y \right\rfloor + \left\lfloor x+3y \right\rfloor$
Từ thành quả ê chứng tỏ $(5m)!(5n)!$ phân chia không còn cho tới $m!n!(3m+n)!(3n+m)!$

Giải:
Đặt $x=a+u$, $y=b+u$ ( a, b là những số vẹn toàn ko âm, $0\leq u,v< 1$)
$BĐT\Leftrightarrow a+b+\left \lfloor 5u \right \rfloor+\left \lfloor 5v \right \rfloor\geq \left \lfloor 3u+v \right \rfloor+\left \lfloor 3v+u \right \rfloor$
Ta tiếp tục CM bất đẳng thức mạnh rộng lớn :
$\left \lfloor 5u \right \rfloor+\left \lfloor 5v \right \rfloor\geq \left \lfloor 3u+v \right \rfloor+\left \lfloor 3v+u \right \rfloor$ (*)
vì u,v với tầm quan trọng như nhau . Không mất mặt tình tổng quát tháo, tao fake sử $u\geq v$
$\Rightarrow \left \lfloor 5u \right \rfloor\geq \left \lfloor 3u+v \right \rfloor$
Nếu $u\leq 2v$ thì $\left \lfloor 5v \right \rfloor\geq \left \lfloor 3v+u \right \rfloor$: $\Rightarrow$ đpcm
Với $u> 2v$ :
Đặt $5u=a^{'}+b^{'}, 5v=c^{'}+d^{'}$ ( $a^{'}, c^{'}$ là những số vẹn toàn ko âm , $0\leq b^{'},d^{'}< 1$)
$(*) \Leftrightarrow a^{'}+c^{'}= \left \lfloor \frac{3a^{'}+c^{'}+3b^{'}+d^{'}}{5} \right \rfloor+\left \lfloor \frac{3c^{'}+a^{'}+3d^{'}+b^{'}}{5} \right \rfloor$ (**)
Vì $1> u> 2v \Rightarrow 5> 5u> 10v \Rightarrow 5> a^{'}+b^{'}> 2c^{'}+2d^{'}\Rightarrow 5> a^{'} \Rightarrow 4\geq a^{'}$
Mà:
$a^{'}+b^{'}> 2c^{'}+2d^{'}\Rightarrow a^{'}\geq 2c^{'}$
( vì thế nếu$a^{'}< 2c^{'}\Rightarrow a^{'}\leq 2c^{'}-1\Rightarrow a^{'}+1-2c^{'}\leq 0, a^{'}+b^{'}-2c^{'}< 0$)
Do đó:
$4\geq a^{'}\geq 2c^{'}$
Kiểm tra (**) so với 9 ngôi trường hợp
( để ý là $3b^{'}+d^{'}< 4, 3d^{'}+b^{'}< 4$):
$a^{'}= 4, c^{'}= 2,1,0$
$a^{'}= 3, c^{'}= 1,0$
$a^{'}= 2, c^{'}= 1,0$
$a^{'}= 1, c^{'}= 0$
$a^{'}=0, c^{'}= 0$
đúng $\Rightarrow đpcm$
Áp dụng:
Theo quyết định lý Legendre( nhằm hiểu biết thêm cụ thể ,chúng ta nhập mục chính phần vẹn toàn của thầy hxthanh), tao chỉ việc CM
$\left \lfloor \frac{5m}{r} \right \rfloor+\left \lfloor \frac{5n}{r} \right \rfloor\geq \left \lfloor \frac{m}{r} \right \rfloor+\left \lfloor \frac{n}{r} \right \rfloor+\left \lfloor \frac{3m+n}{r} \right \rfloor+\left \lfloor \frac{3n+m}{r} \right \rfloor$, với từng $r\geq 2$ (^^)
đặt $m=rm^{'}+x, n=rn^{'}+y$
(trong ê $0\leq x,y< r$ ; r,$m^{'}, n^{'}$ vẹn toàn )
Khi đó
(^^) trở nên :
$\left \lfloor \frac{5x}{r} \right \rfloor+\left \lfloor \frac{5y}{r} \right \rfloor\geq \left \lfloor \frac{3x+y}{r} \right \rfloor+\left \lfloor \frac{3y+x}{r} \right \rfloor$(cmt)
THE END. :icon6:
p/s : Bài với gì sơ sót, hy vọng người xem bổ sung cập nhật thêm thắt :P

Bài viết lách và được sửa đổi nội dung vì thế perfectstrong: 12-02-2012 - 20:49


#13

Đã gửi 13-02-2012 - 12:10

hxthanh

    Tín vật $\sum$

  • Hiệp sỹ
  • 3921 Bài viết

Một số bài xích luyện tự động luyện nhập mục chính phần nguyên

$\boxed{Bt1.6}$
Cho $A=\sqrt{4n^2+n},\; n\in\mathbb{N}$. Chứng minh rằng: $\{A\}\le \dfrac{1}{4}$

(Romania - 2003)


Lời giải:

Ta có: $4n^2 \le 4n^2+n < 4n^2+4n+1$
Suy ra: $\left\lfloor A \right\rfloor = 2n \Rightarrow \left\{ A \right\} = A - \left\lfloor A \right\rfloor = \sqrt{4n^2+n} - 2n$
Bất đẳng thức cần thiết chứng tỏ tương tự với:
$\sqrt{4n^2+n} - 2n \le \dfrac{1}{4};\;(n\in \mathbb{N})$
$\Leftrightarrow 4\sqrt{4n^2+n} \le 8n+1$
$\Leftrightarrow 16(4n^2+n) \le 64n^2+16n+1 $
$\Leftrightarrow 0 \le 1 \Leftrightarrow \text{ true}$


#14

Đã gửi 06-06-2012 - 19:10

hxthanh

    Tín vật $\sum$

  • Hiệp sỹ
  • 3921 Bài viết

Hai bài xích phần vẹn toàn nhập kỳ thi đua của ĐHKHTN trong những năm trước
$\begin{array}{| |}
\hline
&\boxed{\text{Bài 1}}&\\
&\text{Chứng minh rằng với từng } n \text{ bất ngờ thì:}&\\
&\left\lfloor\sqrt[3]{72n+1}\right\rfloor=\left\lfloor\sqrt[3]{9n}\right\rfloor+\left\lfloor\sqrt[3]{9n+1}\right\rfloor=\left\lfloor\sqrt[3]{72n+7}\right\rfloor&\\
&&&\\
\hline
\end{array}$

$\begin{array}{| |}
\hline
&\boxed{\text{Bài 2}}&\\
&\text{Chứng minh rằng với từng số vẹn toàn dương } n \text{ thì:}&\\
&A=n+\left\lfloor\sqrt[3]{n-\dfrac{1}{27}}+\dfrac{1}{3}\right\rfloor^2&\\
&\text{không thể là 1 trong những lập phương đúng!}&
&&&\\
\hline
\end{array}$


#15

Đã gửi 06-06-2012 - 23:14

hxthanh

    Tín vật $\sum$

  • Hiệp sỹ
  • 3921 Bài viết

$\begin{array}{| |}
\hline
&\boxed{\text{Bài 2}}&\\
&\text{Chứng minh rằng với từng số vẹn toàn dương } n \text{ thì:}&\\
&A=n+\left\lfloor\sqrt[3]{n-\dfrac{1}{27}}+\dfrac{1}{3}\right\rfloor^2&\\
&\text{không thể là 1 trong những lập phương đúng!}&
&&&\\
\hline
\end{array}$

Lời giải:
Đặt $m=\left\lfloor\sqrt[3]{n-\dfrac{1}{27}}+\dfrac{1}{3}\right\rfloor=\left\lfloor\dfrac{\sqrt[3]{27n-1}+1}{3}\right\rfloor$
Dễ thấy $m \ge 1, \quad \forall n \ge 1$
Ta có:
$\begin{array}{lrcccl}
& m &\le& \dfrac{\sqrt[3]{27n-1}+1}{3} &<& m+1\\
\Leftrightarrow & 3m-1 &\le& \sqrt[3]{27n-1}&<& 3m+2\\
\Leftrightarrow & 27m^3-27m^2+9m-1 &\le& 27n-1&<& 27m^3+54m^2+36m+8\\
\Leftrightarrow & m^3-m^2+\dfrac{m}{3} &\le& n&<& m^3+2m^2+\dfrac{4m+1}{3}\\
\Leftrightarrow & m^3+\dfrac{m}{3} &\le& n+m^2&<& m^3+3m^2+\dfrac{4m+1}{3}\\
\Rightarrow & m^3 &<& A&<& (m+1)^3\\
\end{array}$
Do $m^3$ và $(m+1)^3$ là $2$ số lập phương liên tục nên $A$ ko thể là số lập phương
$\blacksquare$


#16

Đã gửi 07-06-2012 - 03:12

hxthanh

    Tín vật $\sum$

  • Hiệp sỹ
  • 3921 Bài viết

Để đo lường và tính toán những tổng phần vẹn toàn hữu tỉ, tao với cùng 1 BỔ ĐỀ khá cần thiết sau:
$$\begin{array}{| |}
\hline
\boxed{\text{ BỔ ĐỀ I }}\\
\text{Với từng số vẹn toàn dương } m \text{ và } n \text{ tao có: }\\
\boxed{\sum\limits_{k=0}^n \left\lfloor\dfrac{k}{m}\right\rfloor=\left(n+1-\dfrac{m}{2}\right)\left\lfloor\dfrac{n}{m}\right\rfloor-\dfrac{m}{2}\left\lfloor\dfrac{n}{m}\right\rfloor^2}\\
\hline
\end{array}$$
Chứng minh:
Với $m=1$: Bổ đề trở thành công xuất sắc thức thân thuộc $\sum\limits_{k=0}^n k =\dfrac{n(n+1)}{2}$.
Xét với $m \ge 2$

Ta tiếp tục group những số hạng theo dõi số trật tự đồng dư modul $m$ bằng phương pháp đặt
$k=pm+r,\quad (0 \le r \le m-1)$
Khi ê $0 \le p \le \left\lfloor\dfrac{n}{m}\right\rfloor-1$, phần "dôi ra" tiếp tục chạy kể từ $k=m\left\lfloor\dfrac{n}{m}\right\rfloor $ cho tới $k=n$

Ta có:
$\sum\limits_{k=0}^n \left\lfloor\dfrac{k}{m}\right\rfloor=\sum\limits_{p=0}^{\left\lfloor\frac{n}{m}\right\rfloor-1}\sum\limits_{r=0}^{m-1} \left\lfloor\dfrac{pm+r}{m}\right\rfloor+\sum\limits_{k=m\left\lfloor\frac{n}{m}\right\rfloor}^n \left\lfloor\dfrac{k}{m}\right\rfloor=\sum\limits_{p=0}^{\left\lfloor\frac{n}{m}\right\rfloor-1} mp + \left(n+1-m\left\lfloor\dfrac{n}{m}\right\rfloor\right)\left\lfloor\dfrac{n}{m}\right\rfloor$
$\qquad\qquad = m\left(\dfrac{1}{2}\left(\left\lfloor\dfrac{n}{m}\right\rfloor-1\right)\left\lfloor\dfrac{n}{m}\right\rfloor\right)+(n+1)\left\lfloor\dfrac{n}{m}\right\rfloor-m\left\lfloor\dfrac{n}{m}\right\rfloor^2$
$\qquad\qquad=\left(n+1-\dfrac{m}{2}\right)\left\lfloor\dfrac{n}{m}\right\rfloor-\dfrac{m}{2}\left\lfloor\dfrac{n}{m}\right\rfloor^2$
Vậy tao với điều cần chứng tỏ.
$\square$
Sau phía trên tao tiếp tục vận dụng BỔ ĐỀ này nhằm xử lý Bài toán $\boxed{Bt2.11}$


#17

Đã gửi 07-06-2012 - 04:33

hxthanh

    Tín vật $\sum$

  • Hiệp sỹ
  • 3921 Bài viết

$\boxed{Bt2.11}$

Tính: $S=\sum\limits_{k=1}^n \left\lfloor \dfrac{k^2-3k+2}{5} \right\rfloor$

- Bài này nom nhỏ gọn xinh xắn là thế, tuy nhiên đo lường và tính toán lại chẳng đơn giản và giản dị chút nào!
- Việc thứ nhất là tao rất cần phải "hạ bậc" so với tử thức của phân thức nhập vệt phần vẹn toàn, và tất nhiên tao cần xét theo dõi modul $5$
- Đặt $k=5p+r$ với $0 \le r \le 4$ và làm cho tiện, ngoài cần xét riêng rẽ lẻ $5$ tình huống tao viết lách gộp $r=(0,1,2,3,4)$
hay là $k=5p+(0,1,2,3,4)$
Thay nhập biểu thức nhập vệt phần vẹn toàn tao được:

$\quad\left\lfloor \dfrac{k^2-3k+2}{5} \right\rfloor=\left\lfloor \dfrac{\Big(5p+(0,1,2,3,4)\Big)^2-3\Big(5p+(0,1,2,3,4)\Big)+2}{5} \right\rfloor=$

$=\left\lfloor \dfrac{\Big(25p^2+10p(0,1,2,3,4)+(0,1,4,9,16)\Big)-\Big(15p+(0,3,6,9,12)\Big)+(2,2,2,2,2)}{5} \right\rfloor$
$=5p^2+2pr-3p+\left\lfloor\dfrac{(2,0,0,2,6)}{5}\right\rfloor$
$=5p^2+2pr-3p+(0,0,0,0,1)$
$=5p^2+2pr-3p+\left\lfloor\dfrac{r+1}{5}\right\rfloor$
$=5p^2-4p+2pr+\left\lfloor\dfrac{k+1}{5}\right\rfloor$
Từ ê suy ra:
$S=\sum\limits_{k=1}^n \left\lfloor \dfrac{k^2-3k+2}{5} \right\rfloor=\sum\limits_{1\le k=5p+r \le n} \left(5p^2-4p+2pr+\left\lfloor\dfrac{k+1}{5}\right\rfloor\right)$
$=\sum\limits_{1\le k=5p+r \le n} \left(5p^2-4p+2pr\right)+\sum\limits_{k=1}^n\left\lfloor\dfrac{k+1}{5}\right\rfloor$
$\begin{array}{}
&& && \\
=&\sum\limits_{p=0}^{\left\lfloor\frac{n}{5}\right\rfloor-1}\sum\limits_{r=0}^{4}\left(5p^2-4p+2pr\right)+\sum\limits_{p=\left\lfloor\frac{n}{5}\right\rfloor} \sum\limits_{r=0}^{n-5\left\lfloor\frac{n}{5}\right\rfloor}\left(5p^2-4p+2pr\right)+\left\lfloor\dfrac{n+1}{5}\right\rfloor&+\sum\limits_{k=0}^n \left\lfloor\dfrac{k}{5}\right\rfloor\end{array}$

$=\sum\limits_{p=0}^{\left\lfloor\frac{n}{5}\right\rfloor-1}\left(25p^2\right)+\sum\limits_{r=0}^{n-5\left\lfloor\frac{n}{5}\right\rfloor}\left(5\left\lfloor\dfrac{n}{5}\right\rfloor^2-4\left\lfloor\dfrac{n}{5}\right\rfloor+2\left\lfloor\dfrac{n}{5}\right\rfloor r\right)+\left\lfloor\dfrac{n+1}{5}\right\rfloor+\sum\limits_{k=0}^n \left\lfloor\dfrac{k}{5}\right\rfloor$

$=\dfrac{25}{6}\left\lfloor\dfrac{n}{5}\right\rfloor\left(\left\lfloor\dfrac{n}{5}\right\rfloor-1\right)\left(2\left\lfloor\dfrac{n}{5}\right\rfloor-1\right)+\left(n+1-5\left\lfloor\dfrac{n}{5}\right\rfloor\right)\left(5\left\lfloor\dfrac{n}{5}\right\rfloor^2-4\left\lfloor\dfrac{n}{5}\right\rfloor\right)+ \qquad+2\left\lfloor\dfrac{n}{5}\right\rfloor\dfrac{1}{2}\left(n-5\left\lfloor\dfrac{n}{5}\right\rfloor\right)\left(n+1-5\left\lfloor\dfrac{n}{5}\right\rfloor\right)+\left\lfloor\dfrac{n+1}{5}\right\rfloor+\left(\left(n+1-\dfrac{5}{2}\right)\left\lfloor\dfrac{n}{5}\right\rfloor-\dfrac{5}{2}\left\lfloor\dfrac{n}{5}\right\rfloor^2\right)\quad\text{(Theo Bổ đề I)}$

Khai triển đi ra rồi rút gọn gàng lại thì được:

$\boxed{S=\sum\limits_{k=1}^n \left\lfloor \dfrac{k^2-3k+2}{5} \right\rfloor=\left\lfloor\dfrac{n}{5}\right\rfloor\left(n^2-2n-\dfrac{4}{3}+5\left\lfloor\dfrac{n}{5}\right\rfloor-5n\left\lfloor\dfrac{n}{5}\right\rfloor+\dfrac{25}{3}\left\lfloor\dfrac{n}{5}\right\rfloor^2\right)+\left\lfloor\dfrac{n+1}{5}\right\rfloor}$
_____________________________________________________________
p/s: Nhìn chóng mặt quay cuồng vượt lên trước nhỉ :D
So sánh một số trong những độ quý hiếm của Vế Trái
và một số trong những độ quý hiếm tính được ở Vế Phải


#18

Đã gửi 07-06-2012 - 16:48

hxthanh

    Tín vật $\sum$

  • Hiệp sỹ
  • 3921 Bài viết

$\boxed{Bt2.4}$

Tính tổng $S_n=\sum\limits_{k=1}^n \left\lfloor \sqrt{k}+ \dfrac{1}{2} \right\rfloor$

Lời giải:
(Có nhiều phương pháp để tính được tổng này, ở phía trên tao thực hiện cơ hội tay chân nhất này là phân chia đoạn)

Đặt $m=\left\lfloor \sqrt{n}+ \dfrac{1}{2} \right\rfloor$

Suy ra: $m \le \sqrt{n}+ \dfrac{1}{2} < m+1$ (Thực tế không tồn tại vệt bằng)

$\Rightarrow \left(m-\dfrac{1}{2}\right) \le \sqrt{n} < \left(m+\dfrac{1}{2}\right)$

$\Rightarrow \left(m^2-m+\dfrac{1}{4}\right) \le n < \left(m^2+m+\dfrac{1}{4}\right)$

$\Rightarrow \left(m^2-m+1\right) \le n < \left(m^2+m+1\right)$

Do ê tao phân chia đoạn $[1,n]$ trở nên những đoạn $[i^2-i+1,i^2+i]\;1\le i \le m-1$ đoạn cuối là $[m^2-m+1,n]$

Trên từng đoạn loại $i$, ứng $i^2-i+1 \le k \le i^2+i$ thì $\left\lfloor \sqrt{k}+ \dfrac{1}{2} \right\rfloor=i$

Trên đoạn sau cuối, ứng $m^2-m+1\le k \le n$ thì $\left\lfloor \sqrt{k}+ \dfrac{1}{2} \right\rfloor=m$
Suy ra:
$S_n=\sum\limits_{k=1}^n \left\lfloor \sqrt{k}+ \dfrac{1}{2} \right\rfloor= \sum\limits_{i=1}^{m-1}\sum\limits_{k=i^2-i+1}^{i^2+i} i + \sum\limits_{k=m^2-m+1}^n m$
$S_n=\sum\limits_{i=1}^{m-1} (2i^2) + m(n+m-m^2)$
$S_n=\dfrac{2m(m-1)(2m-1)}{6} + m(n+m-m^2)$

$\boxed{S_n=\sum\limits_{k=1}^n \left\lfloor \sqrt{k}+ \dfrac{1}{2} \right\rfloor=mn-\dfrac{m(m-1)(m+1)}{3},\quad \text{với } m=\left\lfloor\sqrt{n}+ \dfrac{1}{2}\right\rfloor}$


#19

Đã gửi 07-06-2012 - 17:03

hxthanh

    Tín vật $\sum$

  • Hiệp sỹ
  • 3921 Bài viết

Một Việc tương tự động này là bài xích KMO 1997
Tính $S=\sum\limits_{k=1}^n \left\lfloor\sqrt{k}\right\rfloor$

Giải tương tự động như $\boxed{Bt2.4}$ tao với kết quả

$S=\sum\limits_{k=1}^n \left\lfloor\sqrt{k}\right\rfloor = np-\dfrac{p(p-1)(2p+5)}{6};\quad\text{ với }p=\left\lfloor\sqrt{n}\right\rfloor$

Sử dụng thành quả này và thành quả của $\boxed{Bt2.4}$ với quyết định lý Hermite tao dễ dàng và đơn giản xử lý bài

$\boxed{Bt2.5}$

Tính tổng $S_n=\sum\limits_{k=1}^n \left\lfloor 2\sqrt{k} \right\rfloor$

Áp dụng quyết định lý Hermite tao với $\left\lfloor\sqrt{k}\right\rfloor+\left\lfloor\sqrt{k}+\dfrac{1}{2}\right\rfloor = \left\lfloor 2\sqrt{k}\right\rfloor$
Từ ê dễ dàng và đơn giản tính được kết quả!


#20

Đã gửi 07-06-2012 - 22:34

hxthanh

    Tín vật $\sum$

  • Hiệp sỹ
  • 3921 Bài viết

$\boxed{Bt2.6}$
Cho sản phẩm số $\{U_n\}_1^\infty \; :\;\{1,2,2,2,3,3,3,3,3,4,4,4,4,4,4,4,5,...\}$
Được xác lập vì thế quy luật: $1$ số $1$; $3$ số $2$; $5$ số $3$;...;$2k-1$ số $k$;...
Tìm số hạng tổng quát tháo của sản phẩm số bên trên.

(Bài này dễ dàng nhưng mà :P )
Lời giải:
Xét $U_n$ nhập group có mức giá trị $k$, (nhóm này còn có $2k-1$ số hạng)
Số những số hạng phía đằng trước nó với $(k-1)$ group, số thành phần của từng group là những số lẻ liên tục, vì thế đó:
$\Rightarrow \sum\limits_{i=1}^{k-1}(2i-1)<n<\sum\limits_{i=1}^{k-1}(2i-1)+2k$

$\Rightarrow (k-1)^2<n<k^2+1$

$\Rightarrow (k-1)^2+1 \le n < k^2+1$

Xem thêm: Xem bản ghi âm và bản chép lời cuộc gọi cho kênh thoại

$\Rightarrow \sqrt{n-1}< k \le \sqrt{n-1}+1$

$\Rightarrow k=1+\left\lfloor\sqrt{n-1}\right\rfloor$

Vậy số hạng tổng quát tháo của sản phẩm là $U_n=1+\left\lfloor\sqrt{n-1}\right\rfloor,\qquad\forall n\ge 1$
$\blacksquare$


BÀI VIẾT NỔI BẬT